Core Curriculum for Oncology Nursing Part 2 Ch 7-15

Réussis tes devoirs et examens dès maintenant avec Quizwiz!

Which one of the following is a modifiable risk for stomach cancer? A. Diet high in salted and smoked foods B. Gastric polyps C. Previous gastric surgery D. Family history of the disease

A Rationale: Diets which are high in salted and smoked foods, and low in fruit and vegetable consumption is known as a modifiable risk factor for stomach cancer. Gastric polyps are considered a nonmodifiable risk, as is previous gastric surgery, and a family history of the disease. Other modifiable risks include alcohol intake of more than four drinks per day, smoking, and gastric ulcers. Obesity is also associated with gastric cardia cancer.

Which one of the following is an example of adjuvant chemotherapy therapy for colon cancer? A. FOLFOX B. Carboplatin/paclitaxel C. FOLFIRI D. Cetuximab

A Rationale: FOLFOX is a chemotherapy regimen used for colon cancer in both the adjuvant and metastatic settings. CAPEOX, capecitabine, and 5-FU/leucovorin are also often used in the adjuvant setting. Carboplatin/paclitaxel is not a chemotherapy regimen used for colon cancer, but is considered a common chemotherapy regimen used in ovarian and lung cancers. Cetuximab and FOLFIRI are used in the metastatic colorectal cancer setting.

Which one of the following is a risk factor for gastric cancer that is considered modifiable? A. Alcohol B. Family history C. Epstein-Barr virus D. Lynch syndrome

A Rationale: Modifiable risk factors are those within the control of the individual. For example, alcohol use is considered a modifiable risk factor. Individuals cannot control thei

What percentage of pancreatic cancers are classified as adenocarcinomas? A. 95% B. 80% C. 25% D. 5%

A Rationale: Pancreatic adenocarcinoma is the most common form of pancreatic cancer. It accounts for almost 95% of cases and arises from the exocrine pancreas. Pancreatic neuroendocrine tumors are less common but tend to have a better prognosis.

The virus that is most commonly associated with hepatocellular cancer is which one of the following? A. Hepatitis C virus B. Influenza C. Human papillomavirus infection D. Human immunodeficiency virus

A Rationale: Risk factors for hepatocellular cancer include hepatitis C virus (HCV), hepatitis B virus (HBV), cirrhosis, diabetes, and obesity. Influenza is not a known risk factor for malignancy. Human papillomavirus infection (HPV) and human immunodeficiency virus (HIV) are risk factors for anal cancer.

Which one of the following is a type of chemotherapy used for cholangiocarcinoma? A. Gemcitabine B. Methotrexate C. Cytoxan D. Pembrolizumab

A Rationale: Systemic therapies used in adjuvant and advanced settings include fluoropyridamine-based or gemcitabine-based therapies. Pembrolizumab for patients with MSI-H/dMMR for second-line therapy and third- line therapy for patients PD-L1 positive. Cytoxan is not used in the treatment of colon cancer.

Which one of the following patients should undergo genetic testing for mutation? A. A 60-year-old male with breast cancer. B. An 80-year-old female with grade 1, estrogen receptor (ER)-positive, progestogen receptor (PR)-positive breast cancer. C. A 55-year-old female with ductal carcinoma in situ (DCIS). D. A 60-year-old female who is post-menopausal with no family history.

A. A 60-year-old male with breast cancer. Male breast cancer is an indication for referral for genetic evaluation. Women diagnosed with breast cancer at age 50 or under should be given consideration for genetic evaluation, as well as women age 60 and under with triple negative breast cancer. Answers B, C, and D reflect on women with average or older age of onset. These women would not be considered for referral for genetic evaluation unless there was a family history of breast, ovarian, melanoma, pancreatic, prostate, or colon cancer, or possibly a diagnosis of two primary breast cancers.

W. J. is a 56-year-old woman who has just had a mastectomy and was found to have had a positive lymph node. Which one of the following educational points should the nurse perform on the patient's first post-operative visit? A. Avoid injury and trauma to the affected arm B. Avoid upper extremity exercise C. Take temperature and report a temperature of more than 100°F D. Reduce sodium intake to less than 1800 mg daily

A. Avoid injury and trauma to the affected arm. Prevention of lymphedema is far easier than treatment. Prevention measures include measuring the arm for increase in diameter postoperatively as compared to preoperatively. Other prevention measures include avoiding constriction to the arm from tourniquets and blood pressure cuffs as well as trauma or injuries that could lead to infection. Exercise should be supervised and gradually be added into the patient's daily routine. A temperature of greater than 100.5°F could be suggestive of a postoperative infection. Sodium intake has not been shown to impact lymphedema risk.

The protein coding section of a gene is referred to as A. Exon B. Codon C. Intron D. Autosome

A. Exon. Exons are protein-coding segments of a gene. Introns are non-protein-coding segments, the sequence-interrupting piece of a gene. A codon is a sequence of three mRNA nucleotides (e.g., ACG) yielding one (threonine) of the 20 amino acids. An autosome is any chromosome that is not a sex chromosome.

According to the American Cancer Society and the National Cancer Institute, which one of the following is a risk factor for developing a soft tissue cancer? A. Exposure to radiation B. Damaged immune system C. Food allergies D. Exposure to certain viruses

A. Exposure to radiation. According to the American Cancer Society and the National Cancer Institute, exposure to radiation is an identified risk factor for developing cancer of the soft tissue. A damaged immune system is not a risk factor, but a damaged lymph system is an example of a risk factor. Food allergies and exposure to viruses have not been identified as risk factors in developing soft tissue cancers. However, exposure to certain types of chemicals and particular types of family cancer syndromes are examples of risk factors that have been identified.

A sarcoma of the blood vessels is characterized as which one of the following? A. Kaposi sarcoma B. liposarcoma C. rhabdomyosarcoma D. leiomyosarcoma

A. Kaposi sarcoma. Sarcomas of the blood vessels include hemangiosarcoma and Kaposi Sarcoma. Liposarcoma is a sarcoma that starts in the fat cells. Rhabdomyosarcoma and leiomyosarcoma are sarcomas that originate in muscle cells.

Which one of the following is true about Luminal A tumors? A. Luminal A tumors have the highest levels of ER expression: ER positive and/or PR-positive B. Luminal A tumors tend to be high grade C. Luminal A tumors seldom respond to endocrine therapy D. Luminal A tumors have a poor prognosis

A. Luminal A tumors have the highest levels of ER expression: ER positive and/or PR-positive. Luminal A tumors have the highest levels of ER expression: ER-positive and/or PR-positive, these tumors tend to be low grade, are most likely to respond to endocrine therapy, are responsive to chemotherapy, and have a favorable prognosis. Luminal B tumors are typically ER-positive, PR- negative, HER2-positive, and may have an unfavorable subset with aggressive behavior that can be tamoxifen resistant. Basal tumors are negative for ER, PR, and HER2 (triple-negative). They tend to be high grade and often have a poor prognosis; therefore, these tumors will likely benefit from chemotherapy

Diagnostic tests reveal that a patient with limited renal function has developed a primary tumor in the kidney. The nurse should anticipate an order to prepare the patient for which one of the following treatments? A. Partial nephrectomy B. Cytoreductive nephrectomy C. Radiation therapy D. Chemotherapy

A. Partial nephrectomy. Partial nephrectomy is the preferred treatment whenever feasible, especially in patients with limited renal function, bilateral tumors, or a solitary kidney. Cytoreductive nephrectomy is a procedure that may be performed in patients with surgically resectable primary tumor and multiple metastatic sites prior to systemic therapy. Renal cell cancers are unresponsive to radiation therapy. Chemotherapy has not been shown to improve survival in kidney cancer.Partial nephrectomy is the preferred treatment whenever feasible, especially in patients with limited renal function, bilateral tumors, or a solitary kidney. Cytoreductive nephrectomy is a procedure that may be performed in patients with surgically resectable primary tumor and multiple metastatic sites prior to systemic therapy. Renal cell cancers are unresponsive to radiation therapy. Chemotherapy has not been shown to improve survival in kidney cancer.

Mr. S. is a 60-year old man who has been diagnosed with bladder cancer. His nurse has told him of his options for bladder perseveration therapy, including describing what is considered to be the primary treatment modality being currently offered. After hearing the details, Mr. S. is reluctant to undergo the procedure and eventually declines. Which one of the following bladder preservation therapies did the patient most likely decline? A. Radical cystectomy B. External beam radiation therapy C. Trimodality therapy—transurethral resection (TUR) D. Radiation Therapy

A. Radical cystectomy. Radical cystectomy is currently considered the primary treatment modality for bladder preservation but, due to the nature of the procedure, some patients either cannot tolerate the procedure or decline the treatment altogether. Alternatives to radical cystectomy include external beam radiation therapy, radiation therapy, and trimodality therapy—transurethral resection (TUR). Each of these treatments are considered as bladder preservation strategies, though the baseline therapy is currently radical cystectomy.

Individuals from the general population who are enrolled in a clinical trial study to evaluate a new technique for early detection of skin cancer are in which type of the following clinical trials? A. Screening B. Diagnostic C. Quality of life D. Prevention

A. Screening. A screening trial evaluates the effectiveness of new techniques for early detection of cancer in the general population. A diagnostic trial evaluates tests or procedures that may better identify cancer in symptomatic individuals. A quality of life trial explores pharmacologic or non-pharmacologic therapies to minimize cancer related toxicities. A prevention trial evaluates the safety and efficacy of various risk reduction strategies such as chemoprevention or actions such as increasing fruit and vegetable intake, adding exercise, avoiding tobacco, or limiting alcohol use.

Which one of the following patients is most likely to develop breast cancer during their lifetime? A. T. K., who is a 27-year-old white woman. B. J.L, who is a 30-year-old Hispanic woman. C. D.C., who is a 25-year-old white male D. B.R, who is a 37-year-old African American woman.

A. T. K., who is a 27-year-old white woman. T. K, a 27-year-old white woman, has a 1 in 8 chance that she will develop breast cancer in her lifetime. J.L., who is Hispanic, has a lower rate than the other races. Breast cancer is 100 more times likely in woman than in men, so D.C. has the smallest chances of the four choices. Finally, B.R., a 37-year-old African American woman, has a 1 in 10 chance of developing cancer. However, more African American women are diagnosed with breast cancer before the age of 45.

The two most important risk factors that make a person more susceptible to breast cancer include age and which one of the following? A. gender B. race C. reproductive/hormonal factors D. pregnancy

A. gender. There are multiple risk factors for developing breast cancer. Breast cancer is 100 times more common in woman than men. The incidence of breast cancer is higher with aging, only 5% of women before the age of 40 develop breast cancer, but 60% of breast cancer cases occur after age 60 years or older. Breast cancer is 100 more times more common in women than in men. A person's family history also increases risk. Approximately 10% of all breast cancers are due to genetic susceptibility. Pregnancy after the age of 30 or nulliparity is associated with an increased risk.

J.L. is a 63-year-old female patient who has recently been diagnosed with kidney cancer. She is concerned and confused because kidney cancer does not run in her family, and she asks her nurse for an explanation. Her nurse explains that two factors associated with increased risk include which one of the following? A. having a history of non-Hodgkin lymphoma and being overweight B. being of Asian descent and having a history of kidney stones C. consuming processed meats and sedentary occupation D. having a tall stature and being underweight

A. having a history of non-Hodgkin lymphoma and being overweight. Having a history of non-Hodgkin lymphoma and being overweight is the correct answer. The other choices are not correct because they are not known risk factors for kidney cancer, which include lifestyle risk factors such as tobacco use, obesity, and occupational exposure to petroleum products or heavy metals. Dietary risk factors include diets high in fats and protein. Finally, a history of non-Hodgkin lymphoma or sickle cell disease is also considered a risk factor.

K.S. is a 54-year-old female with breast cancer and multiple brain metastases is who scheduled to initiate whole-brain radiation therapy (WBRT). The oncology nurse understands the best description of this treatment's intent is which one of the following? A. palliative B. curative C. adjuvant D. neoadjuvant

A. palliative. WBRT is indicated when there are 4 or more brain metastases and the goal of therapy is palliative (to reduce symptoms and promote comfort care). WBRT is not curative. While pain management and improved or stabilizing neurologic status may be a potential benefit of WBRT, these both fall under

Primary tumors that have been known to spread to the bone include which one of the following? A. prostate B. brain C. leukemia D. melanoma

A. prostate. Metastatic tumors spread to the bone from primary solid tumors. Common tumors include lung, breast, kidney, thyroid, and prostate cancers. Primary cancers that do not spread to the bone include brain cancer, leukemia, and melanoma.

Breast milk is produced by which one of the following? A. terminal duct lobular units B. adipose tissue C. sebaceous tissue D. primary duct units

A. terminal duct lobular units. The terminal duct lobular units (TDLUs) produce breast milk. Adipose tissue becomes more prominent after menopause. Sebaceous tissues are microscopic glands in the skin that secrete an oily substance known as sebum. There are no primary duct units.

What occurs in later stages of immune response with specific immune cells and has memory, which allows for longer-lived immune response called?

Adaptive Immunity

What Aromatase Inhibitor is used in postmenopausal women as primary prevention of high-risk breast cancer?

Aromatase Inhibitors (exemestane)

Which one of the following is part of the diagnostic work-up for esophageal cancer? A. Colonoscopy B. Esophagogastroduodenoscopy C. Brain MRI D. Abdominal ultrasound

B Rationale: Esophagogastroduodenoscopy (EGD) is an endoscopy examination to examine the lining of the esophagus, stomach, and first part of the small intestine. Biopsies may be taken to evaluate for the presence of malignancy. Colonoscopy is a screening test for colorectal cancer. An MRI of the brain and abdominal ultrasound are not part of a routine work-up for esophageal cancer. Other diagnostic modalities for esophageal cancer include a CT or a PET scan of the chest and abdomen, endoscopic ultrasound, and bronchoscopy.

Molecular classification for metastatic colorectal cancer includes which one of the following? A. Programmed death ligand-1 B. KRAS/NRAS C. Estrogen receptor/progesterone receptor D. EGFR mutation

B Rationale: Molecular classification in colon cancer includes KRAS/NRAS (in patients with metastatic disease), BRAF (in patients with metastatic disease), and MMR or MSI. PDL-1 and EGFR are used in the molecular classification of lung cancer. ER/PR (estrogen/progesterone) are used in the molecular classification of breast cancer.

Which of the following is a recommended screening for pancreatic cancer? A. Ultrasound every 6 months is recommended for high-risk individuals B. Routine screening is not recommended for individuals who are not presenting symptoms C. EUS or MRI/magnetic resonance cholangiopancreatography (MRCP) D. No screening methods are currently identified

B Rationale: Routine screening for pancreatic cancer is not recommended for individuals who are not presenting symptoms. EUS or MRI/magnetic resonance cholangiopancreatography (MRCP), however, may be used as a screening technique for individuals with genetic mutations such as HBOC, Lynch syndrome, or PJS. Ultrasound every 6 months is recommended as screening for high-risk individuals of developing hepatocellular cancer, which include patients with all types of cirrhosis and hepatitis B virus (HBV) carriers without cirrhosis. No screening methods have yet to be identified for cholangiocarcinomas - a rare cancer that encompasses all tumors originating from the epithelium of the bile duct.

The approximate number of subjects needed for a Phase I study is which one of the following? A. 10-12 B. 20-100 C. 80-300 D. 500-1000

B. 20-100. On average, 20 to 100 subjects are needed for a Phase I study. Phase I studies often include subjects with many cancer types (e.g., solid tumors), subjects with tumors refractory to standard therapy, and subjects with adequate organ function, specifically bone marrow, liver, and kidney function. Pediatric Phase I studies are conducted after a safety and toxicity evaluation in adults. Ten to twelve subjects are needed for a Phase 0 study, while 80-300 subjects are needed for a Phase II study on average. In contrast, hundreds to thousands of subjects are needed for Phase III and IV studies, on average.

A patient will be undergoing urologic diagnostic testing. Which one of the following nursing interventions should the nurse perform for a radiographic examination of the kidneys, ureter, and bladder (KUB)? A. Assess the patient for history of allergy to iodine dyes or contrast media before performing the test. B. Explain to the patient the need to lie flat on examination table. C. Observe the patient for a reaction to anesthetic or analgesic. D. Monitor the patient for bleeding, and symptoms of a urinary tract infection.

B. Explain to the patient the need to lie flat on examination table.. For a radiographic examination of the kidneys, ureter, and bladder (KUB), the nurse should instruct the patient to let flat on the examination table. Accessing the patient for a history of allergies to iodine dyes or contact media before testing is a nursing intervention instruction for an excretory urography test. For a retrograde urography diagnostic test, nurses are instructed to observe the patient for a reaction to anesthetic or analgesic, and monitor for bleeding, symptoms of a urinary tract infection, dysuria, or difficulty voiding after the test has been completed.

Which one of the following factors makes a patient more likely to have a genetic abnormality based on family history of breast cancer? A. History of multiple breast surgeries B. First-degree relative with breast cancer C. Age of menopause or menarche prior to diagnosis D. Nulliparity

B. First-degree relative with breast cancer. Family history of a first-degree relative having breast cancer is a consideration for genetic evaluation. This is especially important in women with a first degree or second-degree relative with breast cancer diagnosed before the age of 50, multiple family members with breast cancer, or a family history of ovarian cancer. Breast biopsy, reproductive factors such as nulliparity or age at menarche, first pregnancy or menopause are personal, not hereditary, risk factors for developing breast cancer.

Histologic characteristics of breast cancer are often determined by the Bloom-Richardson system. Which statement correctly reflects this histologic classification? A. Grade 1 - low grade and poorly differentiated B. Grade 1 - low grade and well differentiated C. Grade 1 - high grade and well differentiated D. Grade 1 - high grade and poorly differentiated

B. Grade 1 - low grade and well differentiated. In the Bloom-Richardson system or Nottingham grading system, Grade 1 reflects a low grade or well-differentiated breast cancer. Grade 2 represents an intermediate grade and moderately differentiated breast cancer. Grade 3 represents a high grade and poorly differentiated breast cancer.

Which of the following hereditary cancer syndromes is inherited in an autosomal recessive fashion? A. Hereditary retinoblastoma B. MUYTH associated polyposis C. Hereditary diffuse gastric cancer D. Multiple endocrine neoplasia type 1

B. MUYTH associated polyposis. MUTYH-associated polyposis (MAP) is an autosomal recessive hereditary cancer syndrome and is associated with colon cancer and duodenal cancer, as well as colon, duodenal, and gastric fundic gland polyps, osteomas, sebaceous gland adenomas, and pilomatricomas. Polyp counts range from a few to > 1000 with biallelic MUTYH mutations. Hereditary retinoblastoma, hereditary diffuse gastric cancer, and multiple endocrine neoplasia are all inherited in an autosomal dominant fashion. Hereditary diffuse gastric cancer found on the CDH1 gene is associated with diffuse gastric cancer, lobular breast cancer, adenocarcinoma and epithelial ovarian cancer, prostate and signet ring colon cancer. Hereditary retinoblastoma is found on the RBI gene and is associated with malignant tumors of the retina, usually occurring before age 5. A family history of retinoblastoma, bilateral retinal tumors, and multifocal tumors have the highest chance to have hereditary retinoblastoma. Individuals with hereditary retinoblastoma also have an increased risk for pinealoblastoma, osteosarcomas, sarcoma, and melanoma. Multiple endocrine neoplasia type 1 (MEN1) is found on the MEN1 gene. It is associated with endocrine and nonendocrine tumors, including tumors of the parathyroid glands, pituitary gland, and the pancreas.

Which one of the following is a nursing implication when trying to maximize and promote a patient's health and safety after surgery? A. Teaching a patient to manage and identify symptoms, including providing recommendations on when to report symptoms. B. Monitoring vital signs, hemoglobin, hematocrit, kidney function tests, and urine output. C. Teaching patients how to perform coping skills to control anxiety and fear. D. Monitoring patients for signs of distress.

B. Monitoring vital signs, hemoglobin, hematocrit, kidney function tests, and urine output. A recommendation to maximize patient safety postoperatively is monitoring vital signs, hemoglobin, hematocrit, kidney function tests, and urine output. These are critical measures when determining a patient's condition after surgery. Teaching a patient to manage and identify symptoms, including providing recommendations on when to report symptoms, occurs during patient education regarding follow-up care and surveillance, and not as a safety measure after surgery. These are critical measures when determining a patient's condition after surgery.

Which one of the following is a characteristic of an expanded access protocol? A. An investigational drug is used outside of a designated clinical trial. B. Provides a means to use a therapy off label. C. Patient must have early-stage cancer. D. Requires United States Food and Drug Administration approval within 72 hours of use.

B. Provides a means to use a therapy off label. Expanded access provides a means for patients and their physicians to use an investigational drug outside of a designated clinical trial. Off label use of a drug is not a characteristic of expanded access. Expanded access is restricted to patients with a serious condition or disease who no longer have satisfactory medical options available. In compassionate use, not expanded access, approval from FDA may be obtained within 24 hours in emergency situations.

Ewing family tumors (EFT) are associated with: A. distant metastasis B. adolescence C. age over 65 years D. hormonal therapy

B. adolescence. For EFTs, 50% of patients diagnosed are adolescents. Approximately 50% of patients diagnosed are adolescents. EFTs are associated with retinoblastoma and skeletal anomalies. They tend to be highly malignant (approximately 25% with metastases at time of diagnosis to lungs, lymph nodes, other bones). The five-year disease-free survival rate is approximately 73% because of effective multimodality therapies, and precision in surgery. EFTs are characterized as local or regional to the bone. EFTs, a type of bone cancer, are not associated with hormonal therapy.

K. L. is a 47-year-old male patient who has been diagnosed with advanced prostate cancer. He arrives at the physician's office to discuss treatment options with his nurse. Options for this patient include which one of the following? A. radical prostatectomy B. hormonal manipulation C. insertion of radioactive seeds into the prostate D. cryosurgery to freeze the involved prostatic tissue

B. hormonal manipulation. Hormonal manipulation is the accepted standard for treating patients with metastatic prostate cancer. The other options listed - radical prostatectomy, brachytherapy with radioactive seed placement into the prostate, and cryosurgery - are used to treat patients with early-stage prostate cancer.

D.P. is a 48-year-old woman who was treated 2 years ago for HER2-positive breast cancer with an anthracycline and taxane, as well as trastuzumab for 1 year. She calls the clinic stating she is short of breath. She is scheduled for a follow-up visit immediately. Her oncology team is concerned about which one of the following? A. brain and liver metastasis B. lung metastasis and cardiac failure C. liver and lung metastasis D. brain and lung metastasis

B. lung metastasis and cardiac failure. The patient has received trastuzumab, which can lead to a decreased left ejection fraction or pulmonary problems. An anthracycline can also lead to cardiac toxicity. Breast cancer often metastasizes to the lung. Breast cancer also metastasizes to the brain and liver, but her symptoms are not suggestive of metastasis to these organs.

A clinical trial study which explores the results of health care practices and interventions, includes patient-based outcomes, as well as the study of populations, databases, and the delivery of healthcare is described as which one of the following? A. interventional B. outcomes research C. cohort studies D. experimental

B. outcomes research. Outcomes research explore the results of healthcare practices and interventions, and feature patient-based outcomes, as well as the study of populations and different healthcare delivery methods. The clinical trials in experimental studies are designed to answer a different research question. Cohort studies are defined as clinical trial studies where subjects who have no reported outcomes or conditions are followed and compared, based on exposure. In a cross-sectional study, described is the association between a condition and other characteristics that may exist in a specific group.

Mutations in the MSH2 gene are associated with cancers of the A. lung B. ovary C. sarcoma D. thyroid

B. ovary. Lynch syndrome, which was previously known as hereditary nonpolyposis colorectal cancer [HNPCC]), is associated with mutations in the following genes: MLH1, MSH2 (including methylation due to EPCAM deletion), MSH6 and PMS2. Lynch syndrome is characterized by microsatellite instability (MSI) due to defective mismatch repair. Cancers associated with Lynch syndrome include cancers of the colon, rectum, stomach, small intestine, esophagus, biliary tract, brain, endometrium, and ovary. Other cancers at

The small arm of the chromosome is labeled as A. o B. p C. q D. s

B. p. The small arm is known as the petite arm and is labeled as "p". The long arm is labeled as "q" because "q" comes after "p" in the alphabet. There is no "o" or "s" arm.

A common inclusion criteria for eligibility used broadly in oncology clinical trials is termed as which one of the following? A. geographic area of residence B. performance status C. no prior research participation D. number of children

B. performance status. Eligibility criteria are characteristics that potential participants must meet to be enrolled into the trial and includes demographic, disease-specific, and treatment-related variables. These include inclusion and exclusion criteria. Common inclusion criteria that must be satisfied before an oncology patient can enter a trial includes performance status using indicators such as laboratory values, Eastern Cooperative Oncology Group (ECOG) or Karnofsky Performance Status. Other common inclusion criteria include stage and/or status of tumor, presence of measurable disease, and presence or absence of biomarkers. Geography or place of residence is not usually an eligibility criteria, although proximity to the trial may make participation easier. Some studies may not allow previous research participation; this can vary from study to study. The number of children a potential study participant has is seldom an eligibility criteria.

The individual with primary responsibility to ensure the ethical conduct of the research study is which one of the following? A. study coordinator B. principal investigator C. statistician D. data manager

B. principal investigator. The principal investigator ensures the ethical conduct of the research study. The study coordinator, statisticians, and data managers are all important members of the research team, but not responsible for the research study as a whole. The responsibilities of these other research members may vary from study to study.

C. M. is a 47-year-old female patient who has been diagnosed with urothelial carcinoma of the bladder. The nurse educating the patient and her family about this type of bladder cancer explains that urothelial carcinoma of the bladder is: A. the least common type of bladder cancer. B. responsible for most cases of bladder cancer. C. most likely to be invasive at diagnosis. D. associated with changes to chromosome 9.

B. responsible for most cases of bladder cancer. Urothelial carcinoma of the bladder is the most common type of bladder cancer and comprises about 95% of bladder tumors. In 70% to 80% of cases, this type of tumor is generally diagnosed before it has become invasive and invaded the muscle wall of the bladder.

Grading for prostate cancer is based on the Gleason Score, which is determined by results from which one of the following? A. imaging tests using MRI or CT B. tissue specimen examination C. blood tests for PSA levels D. bone marrow aspiration

B. tissue specimen examination. The Gleason Score is based on microscopic examination of the prostate tumor tissue specimen. The pathologist determines the most common cell grade seen in the largest portion of the specimen (the primary cell grade) and in the second largest portion (the secondary cell grade). The two grades are then added together to determine the Gleason score. This score provides information about the aggressiveness of the disease in the prostate and serves as a guide for treatment strategies. Imaging tests using MRI or CT provide information about the extent of the disease but are not part of the process for determining the Gleason score, which is based on characteristics of the malignant tumor cells. PSA levels are not used for computing the Gleason Score but may be useful as a marker for disease progression. Bone marrow aspiration is not used as a screening or diagnostic measure for prostate cancer.

T. C. is a 35-year-old son of a patient who has been diagnosed with bladder cancer. During a recent visit, he asks the nurse for advice on which behaviors he should adopt to help him reduce the risk of him becoming diagnosed with cancer, like his father. The nurse explains that factors associated with increased risk include which one of the following? A. weight loss B. tobacco use C. excessive fluid intake D. diet low in processed meats

B. tobacco use. Tobacco use is the most significant risk factor, accounting for 50% to 66% of all bladder tumors in men and 25% in women. Weight loss is not a risk factor; rather a high BMI may increase risk, and contribute to risk of disease recurrence. Excessive fluid intake does not increase bladder cancer risk but not drinking enough fluids is reported to be a risk factor. Consuming a diet low in processed meats may be somewhat protective for bladder cancer.

Which genetic mutation is associated with hereditary breast cancer?

BRCA1 (1/100 chance), BRCA2 (6/100 chance)

Which one of the following is a significant risk factor for adenocarcinoma of the esophagus? A. Smoking B. Alcohol C. Gastroesophageal reflux disease D. Somatic BRCA1/2 mutation

C Rationale: Gastroesophageal reflux disease (GERD) is a risk factor for adenocarcinoma of the esophagus. A somatic BRCA1/2 mutation is a potential mutation found in a tumor cell and is not a risk factor. Smoking and alcohol use are risk factors for squamous cell carcinoma of esophagus, not adenocarcinoma.

Which one of the following viruses is most commonly associated with anal cancer? A. hepatitis B virus B. Epstein-Barr virus C. Human papillomavirus (HPV) D. Human herpes virus (HHV-8)

C Rationale: Risk factors for anal cancer include human papillomavirus infection (HPV) infections, human immunodeficiency virus (HIV) infection, anal sex, and lowered immunity. Hepatitis B (HBV) is associated with liver cancer. EBV is associated with nasopharyngeal cancer. Human herpes virus (HHV-8) is associated with Kaposi sarcoma.

Based on statistics reported by the National Cancer Institute (NCI) in 2018, between 1975 and 2010, childhood osteosarcoma mortality decreased by more than which one of the following percentages? A. 56 percent B. 66 percent C. 50 percent D. 40 percent

C. 50 percent. Based on statistics reported by the National Cancer Institute (NCI) in 2018, between 1975 and 2010, childhood osteosarcoma mortality decreased by more than 50%. In adolescents ages 15 to 19 years old, the 5-year survival rate was reported to have increased 56% to approximately 66%, so those percentage values do not equal the decrease in mortality. Forty percent is incorrect.

A "3 + 3" Phase I trial to determine maximum tolerated dose is an example of which type of clinical trial design? A. Factorial B. Parallel C. Adaptive D. Basket

C. Adaptive. The "3 + 3" Phase I trial designed to determine maximum tolerated dose is an adaptive design. Adaptive design allows investigators to change trial design without compromising the integrity and validity of the trial. In a 3 + 3 trial, three patients start the trial at a given dose and, if no dose-limiting toxicities are observed, three more patients are added at a higher dose until the first instance of limiting toxicity is observed, then three more patients will be added at the same dose. Dose limiting toxicity in two or all three patients will identify the next lower dose as the maximum tolerated dose. Factorial design allows for multiple factors to be studied simultaneously. Parallel design randomizes participants to one of several treatment groups. Basket trials enroll patients with any cancer type sharing a specific target.

Which one of the following mutations is associated with a hereditary risk for developing lobular breast cancer? A. MUTYH B. BRCA2 C. CDH1 D. PMS2

C. CDH1. Mutations in the CDH1 gene are associated with a 45% lifetime risk of developing lobular breast cancer, as well as an increased risk for developing diffuse gastric cancer. MUTYH mutations are associated with an autosomal recessive condition that places the individual at increased risk for colon polyposis. BRCA 2 mutations are associated with an increased risk for developing breast cancer, but not necessarily lobular breast cancer. PMS2 mutations are part of Lynch syndrome. Individuals with Lynch syndrome are at particularly high risk for developing colon, uterine, ovarian, and other gastrointestinal cancers.

What type of breast cancer is suggestive of poor prognosis? A. ER-positive, PR-positive invasive ductal carcinoma B. ER-positive, PR-positive invasive lobular carcinoma C. ER-negative, PR-negative, Her-2/Neu-negative breast cancer

C. ER-negative, PR-negative, Her-2/Neu-negative breast cancer. Basal cancers, known as triple negative breast cancer, tend to have a worse prognosis. Luminal A tumors have a high ER/PR expression and tend to respond well to endocrine therapy and are often associated with a better prognosis. The prognosis of lobular carcinoma is similar to that of ductal carcinoma. A poorer prognosis is associated with lymph node involvement.

S.L is a 67-year-old women who was recently informed she has stage I ER/PR-positive breast cancer. The patient has a family history of breast, ovarian, and pancreatic cancer. She has an oncotype test which shows a risk score of 24. She asks her nurse what all of this means and the nurse responds with which one of the following statements? A. No risk of recurrence B. Low risk of recurrence C. Intermediate risk of recurrence D. High risk of recurrence

C. Intermediate risk of recurrence. The Oncotype test is a 21-gene assay used to predict the effectiveness of chemotherapy and estimate the chance of recurrence in women with early-stage ER/PR-positive breast cancer. The recurrence score is calculated from gene expression. There is no category of no risk. Low risk is a score of 0 to 17, and the addition of chemotherapy may not be effective. A score of 18 to 31 is considered as an intermediate risk and patients need to consider other factors such as age and comorbidities when deciding whether or not to take chemotherapy. A score of higher than 31 suggests a higher risk of a recurrence, and chemotherapy is usually recommended. The Oncotype test reflects risk of recurrence based on genetic characteristics of the tumor. This female patient may have an extensive family history of cancer and possibly hereditary risk. Oncotype does not provide information about this risk.

T.F. is a 54-year-old female patient who reports episodes of hematuria. The nurse anticipates which one of these diagnostic tests to determine etiology? A. Colonoscopy B. Kidney biopsy C. Intravenous pyelogram D. Blood chemistries

C. Intravenous pyelogram. Intravenous pyelogram is the correct answer. This diagnostic test is commonly used to evaluate patients presenting with hematuria. Colonoscopy visualizes the colon, and is not used to determine the cause of hematuria. The kidney is not biopsied to determine the cause of hematuria, and blood chemistry results would not be diagnostic for hematuria.

R. J. is a 42-year-old male who is complaining of abdominal pain and has mentioned that his stools have been tarry with a foul smell. His nurse should conduct a physical examination with the suspicion that R.J. might be suffering from which one of the following? A. Chondrosarcoma B. Kaposi sarcoma C. Soft Tissue sarcoma D. Osteosarcoma

C. Soft Tissue sarcoma. With complaints of worsening abdominal pain and potential blood in his stools, R. J. is presenting with possible signs of a possible soft tissue sarcoma. There may be worsening abdominal pain due to a retroperitoneal mass. When conducting her examination, the nurse may discover that the mass may or may not be visible, is firm to the touch, nontender, and possibly warm. Many soft tissue lesions can be benign. Chondrosarcoma is a cartilaginous tissue, commonly affecting the pelvis, femur, and shoulder. Kaposi sarcoma usually appears first as legions on the skin and is in the same family as Epstein-Barr virus. Osteosarcoma is commonly found in adolescents and young adults.

A.R. is a 60-year-old woman who arrives at an office visit complaining of bone pain. She explains to her nurse how she is feeling and what the sensation is like. Typically, patients describe bone pain as which one of the following? A. prickly B. intermittent C. aching D. sharp

C. aching. Bone pain is described as dull and aching, increasing at night and increasing over time. Prickly and sharp pain describes pain that involves the nervous system. Bone pain is steady pain, not intermittent pain.

An approved agent that could be utilized for the primary prevention of breast cancer in high-risk post- menopausal women with breast cancer is the use of which one of the following? A. retinoids B. metformin C. aromatase inhibitors D. Cox-2 inhibitors

C. aromatase inhibitors. Aromatase inhibitors can prevent a new breast cancer from developing and are appropriate to utilize in women who are known to be postmenopausal. Premenopausal and postmenopausal women can also consider utilizing tamoxifen to reduce the risk of developing breast cancer. Metformin, retinoids, and cox-2 inhibitors are under investigation to determine if they are effective for the primary prevention of breast cancer.

After amputation, the patient may experience phantom limb pain: A. for 1-2 weeks after surgery. B. postoperatively, starting 8 weeks or more after surgery. C. chronically. D. intermittently when standing.

C. chronically. Phantom limb pain or sensation can occur one to four weeks postoperatively. This type of pain can resolve in a few months or can be chronic. The patient may describe phantom limb pain as itching, pressure, tingling, severe cramping, throbbing, and/or a burning pain. Phantom limb pain occurs over a period of time and not just when the patient intermittently stands.

A clinical trial study where subjects who have no reported outcomes or conditions are followed and compared, based on exposure, is described as which one of the following? A. experimental B. outcomes research C. cohort studies D. observational

C. cohort studies. A clinical trial study where subjects who have no reported outcomes or conditions are followed and compared based on exposure is a cohort study. In an experimental or interventional study, participants receive specific interventions. Each type of clinical trial in an experimental or interventional study is designed to answer a different research question. Studies defined as outcomes research explore the results of healthcare practices and interventions, and feature patient-based outcomes, as well as the study of populations and different healthcare delivery methods.

T. K. is a 43-year-old male patient. During a physical assessment, the nurse notes a palpable mass in the area of a patient's fibia. A key part of assessment and documentation would be to include a(n): A. needle aspiration B. incision and drainage C. comparison to the unaffected side D. evaluation of complete blood count results

C. comparison to the unaffected side. A possible bone or soft tissue mass may or may not be visible or palpable. The mass may be firm, nontender, and warm. Its size should be compared bilaterally to the other fibia. This type of mass is not described as pus-filed or pock-marked. Anemia is a condition related to the patient's blood, not the bone or soft tissue mass.

Diagnostic measures utilized in the diagnosis of breast cancer include which one of the following? A. mammography B. breast MRI C. core needle biopsy D. whole-breast ultrasound

C. core needle biopsy. Diagnostic modalities for breast cancer include core needle biopsy, stereotactic vacuum-assisted breast biopsy, fine-needle aspiration (FNA), incisional biopsy, or excisional biopsy. Imaging with mammography, breast MRI, or breast ultrasound may be used to localize the site to be biopsied.

Potential complications after treatment with radio- active seed placement into the prostate include which one of the following? A. decreased libido B. constipation C. impotence D. anemia

C. impotence. Impotence is the correct answer. Some form of impotence has been seen in 6% to 61% of men following brachytherapy to treat prostate cancer. Diarrhea, rather than constipation, is associated with both radiation therapy and brachytherapy. Anemia is not a side effect of brachytherapy because little bone marrow is exposed to radiation with this treatment.

The clinical trial endpoint of "time from randomization until death" is known as which one of the following terms? A. disease-free survival B. objective response rate C. overall survival D. time to progression

C. overall survival. The definition of overall survival is the time from randomization until the time of death. Disease free survival is the time from randomization until recurrence of tumor or death from any cause. The objective response rate refers to the proportion of patients with

J. L. is a 49-year male with lung cancer who has recently enrolled into a clinical trial. The clinical trial explores new drug therapies to minimize toxicities related to cancer and cancer treatments. The clinical trial is an example of which one of the following? A. screening B. prevention C. quality of life D. therapeutic

C. quality of life. J. L. is participating in a quality of life study. This type of clinical trial explores pharmacologic or nonpharmacologic treatments to minimize toxicities related to cancer and cancer treatments. In contrast, screening trials are meant to evaluate the effectiveness of new techniques for early detection of cancer in the general populous. Diagnostic trials evaluate types of procedures or tests that may better help identify cancer in individuals who present with symptoms of the disease. Treatment or therapeutic trials evaluate the safety of new drugs, vaccines, biological agents, approaches to surgery or radiation therapy, treatment combinations, or other interventions. Even though J. L. is enrolled in a study exploring drug treatments, his trial explores treatments to minimize cancer-related toxicities, rather than exploring the safety and efficacy of new drugs or treatments.

J.C. is a 44-year-old man who presents for his annual well-visit examination and asks his nurse about prostate screening. The nurse informs him that guidelines for prostate-specific antigen (PSA) screening in men at average risk of prostate cancer recommend that screening; A. begins at age 45 years. B. is not useful for men at any age. C. should be done between ages 55 to 69 years. D. once initiated, should be repeated every year.

C. should be done between ages 55 to 69 years. The correct answer is that UAU guidelines indicate that PSA screening between ages 55 to 69 years provides the greatest benefit. Therefore, it is not correct that results of PSA screenings are not useful for men at any age. Routine screening in men 40 to 54 years at average risk is not recommended. Men who decide to initiate PSA screening should have repeat screenings at intervals of 2 or more years.

The goal for the surgical treatment of bone/soft tissue malignancies includes which one of the following? A. amputation B. altered function C. survival D. removing the blood supply

C. survival. The goals of surgical treatment include the following: survival, removal of the tumor, and preserving functionality. Amputation may be the chosen treatment, but it is not the goal of treatment. Removing the malignancy's blood supply is not a surgical goal of treatment.

Which genetic mutation increases risk 45% for Lobular Breast Cancer?

CDH1

What monoclonal antibody blocks ligand binding to EGFR; used in EGFR positive patients to reduce primary tumor size and lymphatic spread?

Cetuximab

What is the class of anticancer therapy called that targets PDL1?

Checkpoint Inhibitors

Which one of the following items is a screening technique for colorectal cancer? A. Abdominal CT B. Abdominal ultrasound C. Pelvic CT D. Immunochemical fecal occult blood test

D Rationale: A CT of the abdomen, abdominal ultrasound, and a CT of the pelvis are all staging and diagnostic procedures used in the evaluation of colon cancer, often following a positive biopsy. Screening tests for colon cancer include guaiac-based stool testing, fecal immunochemical test, immunochemical fecal occult blood test (iFOBT), barium enema, flexible sigmoidoscopy, colonoscopy, or CT colonography.

Which one of the following is a potential curative therapy for hepatocellular carcinoma? A. Transarterial chemoembolization B. Nivolumab C. Sorafenib D. Liver transplant

D Rationale: Local-regional therapies include RFA, TACE, DEB-TACE, and TARE. Nivolumab and Sorafenib are palliative therapies. Liver transplant is a potentially curative option for early-stage HCC.

T. J. is a 41-year-old male, in good health, but whose family history of colon cancer has made him concerned about the possibility of developing colorectal cancer. Which one of the following should T.J's nurse recommend to him as a means of cancer prevention? A. Treat H. pylori, gastric ulcers B. Treat GERD C. Limit exposure to cancer-causing chemicals D. Limit alcohol intake—fewer than two drinks/day

D Rationale: The nurse should recommend to T. J. that he should limit alcohol intake to fewer than two drinks per/day as a course of action to prevent colorectal cancer. The recommendation is two drinks per/day for a male, and one drink/per day for a female. Treating H. pylori, gastric ulcers is a recommendation for the prevention of stomach cancer. Treating GERD and/or Barret's esophagitis is a prevention recommendation for esophageal cancer, while it is recommended to limit exposure to cancer-causing chemicals forhepatocellulaercancer.

Which one of the following depicts a factorial design? Randomization to treatment: A. A or B. B. A→outcome→B. C. A, B, A and B. D. A, B, A and B, placebo.

D. A, B, A and B, placebo. The example of "A, B, A and B, placebo" represents a factorial design. Factorial design allows for multiple factors, such as multiple treatments, to be studied simultaneously. The example of "A or B" represents a parallel design. In parallel design, a participant is randomized to one of several treatment groups. The example of "A→outcome→B" represents a crossover design. A crossover design allows participants to receive more than one treatment. A, B, A and B requires placebo (neither A nor B) to be added to represent a factorial design.

The code of ethics and conduct that focuses primarily on beneficence, respect for persons, and justice is known as which one of the following? A. Nuremberg Code B. Declaration of Helsinki C. Common Rule D. Belmont Report

D. Belmont Report. The Belmont Report focused primarily on beneficence, respect for persons, and justice. The Nuremberg Code focused on voluntary consent. The Declaration of Helsinki focused on informed consent, therapeutic versus non-therapeutic research, and surrogate decision making. The Common Rule, also known as the Protection of Human Research Subjects, focused on informed consent and institutional review boards.

M. K. is a 59-year old male patient who has been diagnosed with clear-cell carcinoma of the kidney. The nurse caring for him is aware that which one of the following is true about this type of kidney cancer? A. The kidney cancer is an unusual type B. The kidney cancer is a type of tumor arising in the renal pelvis C. Cancer of the kidney has the worst prognosis D. Kidney cancer accounts for the majority of cases

D. Kidney cancer accounts for the majority of cases. Clear cell carcinoma is a common type of kidney cancer because it comprises 70% to 80% of all kidney cancer cases. Clear cell carcinoma is thought to arise in the prproximal renal tubule and is not a tumor of the renal pelvis, which are considered to be very rare.

Adjuvant radiotherapy can be a component of treatment in soft tissue tumors: A. only before surgery. B. when the tumor has metastasized. C. as the primary standard of care. D. after the tumor has been debulked.

D. after the tumor has been debulked. Soft tissue tumors can be radiosensitive and radioresponsive. So, adjuvant radiotherapy can be a component of treatment before or after surgery, when the tumor is localized or after the tumor has been surgically debulked or removed. Radiotherapy is not a treatment for distant metastatic spread of disease. For soft tissue tumors, radiotherapy is an additional modality of treatment and not the primary treatment or considered a standard of care.

H.R. is a 67-year-old male patient. He arrives at his local infusion center to receive treatment for kidney cancer. Treatment options that have been shown to improve response rates for this type of disease include use of which one of the following? A. systemic radiation therapy B. antibody-drug conjugates C. cytotoxic chemotherapy D. immunotherapy agents

D. immunotherapy agents. The correct answer is immunotherapy agents. These agents, such as interleukin-2 and interferon- alpha, have produced response rates of 10% to 15% when used as single agents for treating cancer of the kidney. Systemic radiation therapy, in which radioactive drugs are delivered either orally or intravenously, is not used to treat renal cell cancers which are not radiation sensitive. Antibody-drug conjugates, in which an antibody is linked to a cytotoxic agent, are not used to treat kidney cancer because the malignant cells do not express specific antigens that would be the target of antibodies. Cytotoxic chemotherapy has not been shown to improve survival.

Post-reconstruction management of a bone or soft tissue tumor includes a focus on which one of the following? A. thrombocytopenia B. pruritis C. cytokine response D. infection

D. infection. Issues of reconstruction include post-surgery union of non-malignant tissue, infections, healing, and functional concerns (especially with limb salvage). Thrombocytopenia is a condition of decreased platelets (clotting blood cells). Pruritis is an allergic reaction to topical or systemic treatment. For some targeted systematic treatments, cytokine response is when cytokines and other inflammatory mediators are released.

A clinical trial study where participants are not assigned to a specific intervention and health care outcomes are assessed is described as which one of the following? A. experimental B. interventional C. expanded access D. observational

D. observational. In an observational study, participants are not assigned to a specific intervention and health care outcomes are assessed. In an experimental study, participants receive specific interventions and each type is designed to answer different research questions. An interventional study is another name for experimental as described above. Expanded access occurs when a clinical research study provides a means for patients to receive an investigational drug outside of a designated clinical trial. In expanded access, the investigational agent is restricted to patients with a serious condition or disease who no longer have satisfactory medical options available and who may benefit from the investigational therapy.

Chondrosarcoma is best characterized as: A. originating in the lymph nodes. B. originating in osteoid tissue. C. a pediatric disease. D. occurring in middle age (50-60 years old).

D. occurring in middle age (50-60 years old). Chondrosarcoma originates in the cartilage and typically occurs in the pelvis, upper legs, and shoulders. Chondrosarcoma is commonly diagnosed in patients who are 50 to 60 years old, and it is not considered a pediatric disease. Lymphangiosarcoma originates from the lymph nodes. Osteosarcoma originates from osteoid tissue.

What increases the risk of cancer development?

Deficiencies in T-lymphocytes, T Helper cells and Natural Killer Cells

A developmental regulatroy progran/process that causes epithelial cells to lose cell polarity and cell-cell adhesion and have invasive properties is called what?

Epithelial Mesenchymal Transition (EMT)

What is considered the most important trigger of angiogenesis?

Hypoxia

Our body's response against foreign substances is called?

Immune System Response, which involves innate and adaptive immunity.

What are the steps of Carcinogensis?

Initiating mutation, Acquisition of genomic instability, Acquisition of cancer hallmarks, Further genetic evolution.

What is the immediate, first line of defense against invasion; does not rely on previous exposure or memory to b initiated?

Innate Immunity

What are the two most frequent sites of metastasis?

Liver and Lungs

Through a process called ________, which includes complex signaling involving proteases, cytokines, and motility factors, tumor cells are able to migrate through the ECM and gain access to the vascular basement membrane.

Locomotion

Which genetic mutation is associated with hereditary nonpolyposis Colorectal Cancer?

Lynch Syndrome

What is the 70-gene array used to identify women with early-stage breast cancer (either hormone receptor-positive or hormone receptor-negative) who are at risk for distant recurrence after surgery is called.....

MammaPrint, recurrence score is assigned either high or low risk, no intermediate.

What is a 21-gene assay used to predict chemotherapy benefit and estimate the 10-year risk of distant recurrence in women with early-stage, nodenegative, estrogen receptor-positive (ER +) invasive breast cancer called?

Oncotype DX

Cancer cells can evade the T-cell response by expressing ______ to block the T-cell recognition fo the cancer cell that would initiate cytotoxic response.

PDL1

What gene normally inhibits cell division? This gene is mutated in childhood retinoblastoma and many lung, breast, and bone cancers...

RB Gene

What is another primary prevention of breast cancer in high-risk individuals used in women but not men?

Raloxifene

What is the preferred method of axillary lymph node staging?

Sentinel Lymph Node Biopsy

Which genetic mutation is associated with Li-Fraumeni Syndrome?

TP53

What is used as primary prevention of breast cancer in high-risk individuals for men and women?

Tamoxifen

Cancer cells contain high amounts of this enzyme that adds protective telomeres and prevents the telomere segment from shortening, which is believed to allow continued cell replication; contributes to immortalization- cells avoid or survive crisis.

Telomerase

Normal cells have limited amounts of this enzyme. This enzyme adds telomere repeat segments to the ends of chromosomes, which shorten with repeated duplication is called?

Telomerase

Cancer cells use glycolysis for energy production, even in the presence of oxygen which is called?

The Warburg Effect

A example of a chemical released by white blood cells in response to inflammation is?

Tumor Necrosis Factor

The normal, as opposed to the mutant, gene is what?

Wild type gene

According to the Bloom-Richardson or Nottingham grading system, Grade 3 is...

high grade ro poorly differentiated

While using the Oncotype DX assay to predict chemotherapy benefits and risk of recurrence, a recurrence score of > 32 is considered...

high risk

According to the Bloom-Richardson or Nottingham grading system, Grade 2 is...

intermediate grade or moderately differentiated

While using the Oncotype DX assay to predict chemotherapy benefits and risk of recurrence, a score of 18-31 is considered...

intermediate risk

According to the Bloom-Richardson or Nottingham grading system, Grade 1 is...

low grade or well differentiated

While using the Oncotype DX assay to predict chemotherapy benefits and risk of recurrence, a score of 0-17 is considered...

low risk

A tumor is formed from a single precursor cell with genetic alterations that undergoes clonal expansion. Clonal evolution is the process of cells within a tumor accumulating genetic changes over time that are different from one cell to the next. Thus, a tumor may be heterogeneous and consist of cells that arose from the same mother cell that are genotypically different from one another. In this model, which is the third phase in the process of cell mutation? A. Acquisition of cancer hallmarks B. Acquisition of genomic instability C. Initiating mutation D. Further genetic mutation

A. Acquisition of cancer hallmarks. The acquisition of cancer hallmarks is classified as the third phase of the process of cell mutation. A tumor is formed from a single precursor cell with genetic alterations that undergoes clonal expansion. Clonal evolution is the process of cells within a tumor accumulating genetic changes over time that are different from one cell to the next. Initiating mutation is considered the first step in the process, acquisition of genetic instability is next, and, finally, the cell undergoes further genetic mutation.

Which of the following statements explains the derivation of a cancerous tumor? A. Genetic mutations in genes that control cell growth and proliferation are commonly associated with the development of cancer. B. Proto-oncogenes are frequently associated with the proliferation and development of malignancies. C. Passenger mutations are essential to cancers caused by driver mutations. D. Mutations in a DNA repair gene are not associated with environmental carcinogens or inheritance.

A. Genetic mutations in genes that control cell growth and proliferation are commonly associated with the development of cancer. A malignant tumor is derived from genetic instability and genetic mutations in genes that control cell growth and proliferation. B is incorrect as proto-oncogenes are normal genes essential for normal cell growth and proliferation. Mutations occurring in proto-oncogenes convert to oncogene activation to cause uncontrolled cell division. C is incorrect because driver mutations offer a selective growth advantage to cancerous cells, while passenger mutations do not. D is incorrect as mutations in DNA repair genes may be inherited from a parent or acquired over time due to aging or impact of carcinogens from the environment.

Which of the following diseases does not increase the risk for a primary brain tumor? A. Huntington disease B. Neurofibromatosis type 1 C. Li-Fraumeni syndrome D. von Hippel-Lindau disease

A. Huntington disease. All the listed are genetic diseases, but the only one that is not associated with risk of developing brain cancer is Huntington's disease. Neurofibromatosis type 1 is associated with malignant peripheral nerve sheath tumors, optic gliomas, meningiomas, hamartomatous intestinal polyps other gliomas, and leukemias. Li Fraumeni syndrome is associated with soft tissue sarcoma, osteosarcoma, pre-menopausal breast cancer, brain tumors, adrenocortical carcinoma (ACC), and leukemias. Von Hippel Lindau disease is associated with renal cancers, pancreatic neuroendocrine tumors, hemangioblastomas, and pheochromocytomas.

Which of the following groups are not included in the Genetic Non-Discrimination Act (GINA)? A. Native Americans B. African Americans C. Jewish Americans D. Southern Baptist Association

A. Native Americans. The Genetic Information Nondiscrimination Act (GINA), federal legislation enacted in 2008, applies to health insurance and employment discrimination based on genetic information. GINA does not apply to active duty military personnel, Veterans Administration, or Native American Health Service because the laws amended for GINA do not apply to these groups. Health insurance protections with GINA include protections against accessing an individual's genomic information, requirements for an individual to undergo a genetic or genomic test, and using genomic information against a person during medical underwriting. Employment protections include prohibiting employers from accessing an individual's genetic information, use of genomic information to deny employment, or collecting genomic information without consent. GINA does not supersede state legislation that provides for more extensive protections.

Which one of the following is the most abundant granulocyte? A. Neutrophils B. Basophils C. Eosinophils D. Macrophages

A. Neutrophils. Granulocytes have granules in cytoplasm with enzymes that aid in digestion of foreign particles (phagocytosis) and cause inflammation. Neutrophils are the most abundant granulocytes, but they only live about 6 hours. Neutrophils cause inflammatory response due to engulfing and destroying foreign particles and debris. Basophils have IgE receptors that are involved in allergic responses and cause release of histamine and prostaglandins. Eosinophils attack parasites and secrete cytokines that cause inflammation during allergic responses. Macrophages rapidly recognize, ingest, and kill microbes; macrophages are not granulocytes.

K.T., a 29-year old female patient with breast cancer, has requested information on genetic and molecular testing. Her oncology nurse knows that her patient will need more information when she is heard saying: A. Please expedite the results. I need them in 2 days for my second opinion. B. I understand that my results may have implications to the rest of my family. C. I will need to inquire if my health insurance will cover the costs related to genetic and molecular testing. D. There are known risks to my privacy and confidentiality.

A. Please expedite the results. I need them in 2 days for my second opinion. Genetic testing results may take up to 8 weeks and cause waiting-related patient anxiety when treatment is dependent on results. Germline genetic testing has implications for both the patient and family members. It can be expensive and insurance preauthorization is often required. Informed consent issues include how privacy will be maintained and use of the specimen after testing is completed

K.T. is a 29-year old female patient, who has been newly diagnosed with breast cancer. K.T. has asked her oncology nurse why the oncologist would require the result of the biomarker test to determine the treatment plan. Which of the teach-back responses indicates that the patient did not fully understand her oncology nurse's explanation? A. The results will tell us if I need surgery. B. The results can predict if the treatment will work for me. C. My doctor will use the results to find out if I will need chemotherapy after surgery. D. The results can be used to measure how aggressive my cancer is.

A. The results will tell us if I need surgery. Biomarkers can determine disease severity and outcomes and aids in treatment planning. A biomarker is a molecule found in blood, tissues, or other body fluids that signals the presence of a condition or disease; can be used to evaluate response to treatment. Predictive biomarkers provide information on the effect of a therapeutic intervention. A prognostic biomarker biomarker provides information about the patient's overall cancer outcome, regardless of therapy. Biomarkers are not generally used to determine if a patient is a candidate for surgery. A germline mutation may suggest the need for risk reducing surgery.

A patient with a family history of breast cancer has had genetic testing with "23 and Me" and reports her result was "negative". Which of the following information should be discussed with her? A. This tests for BRCA1/2 genetic testing only and includes three mutations. B. Breast cancer predisposition testing from this test is not United States Food and Drug Administration approved. C. Direct-to-consumer testing is approved for use in diagnosis and treatment of breast cancer. D. All direct-to-consumer testing is the same.

A. This tests for BRCA1/2 genetic testing only and includes three mutations. This test includes only the three BRCA1/2 (selected variants) mutations most common in persons of Ashkenazi descent. These are the most common out of thousands of mutations which are not included in the direct-to-consumer test. B is incorrect as it is FDA approved. C is incorrect as it is not FDA approved for diagnosis or clinical decis

Cytokines function to do which one of the following? A. assist with cell signaling during immune responses B. recognize, ingest, and kill microbes C. produce antibodies against antigens after exposure D. supply tissue nourishment, improve oxygenation, and modulate blood viscosity

A. assist with cell signaling during immune responses. Cytokines are proteins that assist in communication between cells of the immune system to aid in rapid response. Macrophages are immune cells of the innate immune system that release cytokines to produce inflammatory response and present antigens to T cells. Plasma cells are differentiated B cells, which produce one specific antibody against a specific antigen as part of the humoral immune response. Erythrocytes or red blood cells develop in the bone marrow and transport oxygen to the body's tissues.

A new oncology nurse is reviewing a patient's medical record and requests help in understanding the process of carcinogenesis. A seasoned nurse explains that the process is due to which one of the following? A. clonal evolution B. convergent evolution C. coevolution D. perseverance

A. clonal evolution. Clonal evolution describes the process of cells within a tumor accumulating genetic changes over time that are different from once cell to the next, A tumor may be varied and consist of cells that rose from the same mother cell that are genotypically different from one another, and arise from the survival of the fittest collection of cancer cells. Convergent evolution and changes and coevolution are two of the six important patterns of macroevolution, which are not involved in the development of carcinogenesis.

Adaptive immunity differs from innate immunity in that adaptive immunity: A. creates memory cells for a longer lived immune response B. uses humoral immunity through T-cell activation C. uses feedback inhibition mechanisms to control tissue damage due to inflammatory response D. uses nonspecific processes for immune defense

A. creates memory cells for a longer lived immune response. Only the adaptive immune system has memory. The innate immune system is a rapid cellular response to invasion of pathogens and/or tissue damage and does not rely on previous exposure to initiate an immune response. Humoral immunity uses antibodies produced by B lymphocytes, which respond to a specific antigen. Cell-mediated immunity uses T lymphocytes to active immune responses. Inflammatory response is controlled by the innate immune system, not adaptive immune system. Nonspecific processes for immune defense is related to innate immunity.

Which mutation-identifying technique will determine the number of mutations in chromosomes? A. cytogenetics B. Sanger sequencing C. genome-wide association studies (GWAS) D. microarray

A. cytogenetics. Cytogenetic reports include modal number of chromosomes, sex chromosome designation; abnormality abbreviation - first chromosome separated with a semicolon from the second chromosome, then the arm and band number B is incorrect as this detects sequence changes in regions being analyzed. Sanger sequencing is a form of gene sequencing that determines the sequence of a gene being tested and detects sequence changes in regions being analyzed. A limitation of this testing is that it may miss mutations outside the coding region or mutations that has a large

The primary tumor that most frequently metastasizes to the brain is which one of the following? A. non-small cell lung cancer B. prostate cancer C. liver cancer D. colorectal cancer

A. non-small cell lung cancer. Non-small cell lung cancer (NSCLC) accounts for 85% of lung cancer diagnoses and is also the most common primary tumor metastasizing to brain, with about 9% of patients with NSCLC developing brain metastases. Prostate cancer commonly metastasizes to the adrenal gland, bone, liver, and lung. Liver cancer is often the site of metastatic cancer. Colorectal cancer typically metastasizes to the liver, lung, and peritoneum.

The concept of immune surveillance occurs: A. when innate and adaptive immunity destroy clinically unmeasurable tumors B. when a rare cancer clone requires resistance to innate immunity but is stabilized by the adaptive immune system C. when tumors enter the equilibrium phase of the proposed tumor suppression mechanism D. as a result of receiving multiple, sequential chemotherapeutic agents

A. when innate and adaptive immunity destroy clinically unmeasurable tumors. Innate and adaptive immunity is the elimination phase of extrinsic tumor suppression mechanism, where the host's immune system responds to microscopic invasion by destroying circulating tumor cells. This is a homeostatic process ongoing in host's with an intact immune system. B is an example of the equilibrium phase of tumor suppression where the first line of defense with natural barriers and inflammatory response have been overcome. Humoral and cell-mediated immunity mechanisms are now holding tumor growth in check. C is incorrect as the equilibrium phase occurs once resistant tumor clones escape immune surveillance and innate immunity has been overcome. D is incorrect as the mechanism of action with chemotherapeutic agents is cell death through cytotoxic cellular processes that inhibit mitosis. Immunotherapy agents function to increase immune surveillance through modification of the adaptive immune system.

What are the two basic types of mutation?

Acquired & Germline

A genomic event that is potentially responsive to a targeted therapy; biological molecules or processes that can be targeted by an existing or experimental drug?

Actionable mutation

A "variant of unknown significance" (VUS) has which of the following characteristics: A. It is identified when a cancer risk has been established. B. A VUS identifies a genetic change in which the association with cancer risk cannot be established. C. Once identified, a VUS maintains that label forever. D. A VUS is an uncommon genetic finding.

B. A VUS identifies a genetic change in which the association with cancer risk cannot be established. A VUS is a change in the genetic material for which there is not enough data to determine if it is a harmful or harmless change in the genetic material. A is incorrect as a VUS is identified when a cancer risk has NOT been established. C is also incorrect as a VUS label can change to pathogenic or benign based on information identified in new types of genetic testing. D is incorrect as a VUS can be a fairly common finding.

Which is the following is an example of pharmacokinetics? A. Examination of how a drug will affect an individual patient B. Examination of how an individual patient will affect a drug C. Study of how a patient's genomes affect responses to certain types of medications D. Study of heredity and the variation of inherited characteristics

B. Examination of how an individual patient will affect a drug. The correct answer is B. Pharmacokinetics examines how the individual will affect the drug. This is contrary to pharmacodynamics, which is how the drug will affect the individual (Answer A). Pharmacogenomics (Answer C) is the study of how a patient's genomes affect responses to medications. And genetics is the study of heredity and the variation of inherited characteristics.

Which of the following is a definition of gene deletion? A. Heritable change that does not alter the DNA sequence but changes gene expression. B. Loss of all or part of a gene found in cancer and other genetic diseases. C. Increase in the copies of a protein made from a gene that may play a role in cancer development. D. Increase in the number of copies of a gene that may cause cancer cell growth or resistance to anticancer drugs.

B. Loss of all or part of a gene found in cancer and other genetic diseases. The correct answer is B. Gene deletion is defined as the loss of all or part of a gene found in cancer and other genetic diseases is defined. Answer A is epigenetic alteration, which is defined as a heritable change that does not alter the DNA sequence but changes gene expression. Gene overexpression (answer C) is the increase in the copies of a protein made from a gene that may play a role in cancer development. Answer D is gene amplification, which is defined as the increase in the number of copies of a gene that may cause cancer cell growth or resistance to anticancer drugs.

The oncology nurse is caring for a 29-year-old female patient newly diagnosed with bilateral breast cancer. Which of the following would raise suspicion for hereditary breast and ovarian cancer (HBOC) syndrome? A. Maternal aunt and paternal uncle with a history of brain cancer at unknown ages. B. Mother was diagnosed with ovarian cancer at age 33 years. C. Brother was diagnosed with testicular cancer at age 25 years after he served in the military during Vietnam. D. 4-year-old nephew diagnosed with leukemia.

B. Mother was diagnosed with ovarian cancer at age 33 years. HBOC is associated with ovarian cancer especially when it occurs before age 50. HBOC is also associated with early onset breast cancer (before age 50) especially when there are multiple family members diagnosed with breast cancer, triple negative breast cancer diagnosed before age 60 or a family history of pancreatic cancer or metastatic prostate cancer. A is incorrect because these individuals are each from both maternal and paternal lineages. C is also incorrect, because even though her brother is young, testicular cancer is not associated with HBOC and testicular cancer is usually diagnosed in younger men. D is incorrect because childhood leukemia is not associated with HBOC.

Which one of the following is a definition of carcinogenesis? A. The creation of new blood vessels from existing ones to provide nutrients and remove waste products. B. The transference of normal cells into cancer cells through a complex and dynamic process that starts with mutations in regulatory cells and is promoted by genomic instability, inflammation, and interactions within the tumor microenvironment. C. The process which causes epithelial cells to lose cell polarity and cell-cell adhesion and have invasive properties so that they can become mesenchymal cells. D. A mechanism that may change the activity of a gene without changing the sequence of DNA.

B. The transference of normal cells into cancer cells through a complex and dynamic process that starts with mutations in regulatory cells and is promoted by genomic instability, inflammation, and interactions within the tumor microenvironment. During carcinogenesis, normal cells are transferred into cancer cells through a complex and dynamic process that starts with mutations in regulatory cells and is promoted by genomic instability, inflammation, and interactions within the tumor microenvironment. Angiogenesis is the creation of new blood vessels from existing ones to provide nutrients and remove waste products. Normal cells have a developmental regulatory program called epithelial mesenchymal transition (EMT). This process causes epithelial cells to lose cell polarity and cell-cell adhesion and have invasive properties so that they can become mesenchymal cells. This process is involved in mesoderm formation and neural tube formation during embryogenesis. It has also been found to play a role in wound healing and organ fibrosis Epigenetics describes a mechanism that may change the activity of a gene without changing the sequence of DNA.

An oncology nurse is caring for a newly diagnosed patient with osteosarcoma who has "skip metastasis". The nurse is aware that this occurs when A. cells bypass one organ and metastasize in another B. cells bypass the first node and reach more distant sites C. cells spread into nearby capillary beds or pulmonary arteriovenous shunts D. tumor cells embed into distant arteries

B. cells bypass the first node and reach more distant sites. "Skip metastasis" is an example tumor cell dissemination throughout the lymphatic system. "Skip metastasis" occurs when cells bypass the first lymph node and reach more distant sites. Tumors bypassing one organ and metastasizing in another is not an example of a pathway in which tumor cells disseminate. Tumor cells spreading through pulmonary capillary beds or pulmonary arteriovenous (AV) shunts is an example of arterial spread. Arteries have thick walls, and are not able to be penetrated as veins are.

According to 2018 statistics from the American Cancer Society, colorectal cancer is the third leading cause of cancer-related deaths in men and in women, and the second most common cause of cancer deaths when men and women are combined. Mr. P. is a 55-year-old man who is has been diagnosed with metastatic colon cancer. The nurse caring for Mr. P. is aware that the signaling pathway that may be playing a role in his metastases is: A. vascular endothelial growth factor B. epidermal growth factor receptor C. medial growth factor D. nerve growth factor

B. epidermal growth factor receptor. Epidermal growth factor receptors participate in colon cancer development and play a role in some colon cancer metastases. Vascular endothelial growth factors may cause tumor cells to spread to regional lymph nodes. Nerve growth factors are primarily involved in the regulation of growth, maintenance, proliferation, and survival of certain target neurons, especially those that transmit pain, temperature, and touch sensations. There is no medial growth factor.

Which of the following information is more important in the application of precision medicine in a patient with a cholangiocarcinoma? A. 11.5 x 10.0 x 9.5 cm right hepatic lobe mass B. high positive expression of programmed death-ligand 1 (> 50%) C. tolerated adjuvant therapy with leucovorin, fluorouracil, and oxaliplatin (FOLFOX) D. alanine aminotransferase 80 U/L, aspartate aminotransferase 123 U/L, total bilirubin 2.9 mg/dL

B. high positive expression of programmed death-ligand 1 (> 50%). Precision medicine is the use of specific information about a person's genes, proteins, and environment to prevent, diagnose, and treat disease. Tumor size, treatment history, and laboratory values may affect response but are not the primary consideration when applying precision medicine. PD-L1 expression is an example of using genomic information from the tumor and impacts treatment decisions in head and neck squamous cell, Hodgkin lymphoma, Merkel cell carcinoma, urothelial carcinomas. Targeted therapies that might be utilized in those with high PD-L1 expression include atezolizumab, avelumab, durvalumab, nivolumab, and pemrolizuma

Application of the Central Dogma of molecular biology to oncology nursing practice is important because: A. canines have the same DNA nucleotides as humans. B. it explains the production of protein for all body functions. C. DNA and RNA contain identical nucleic acids for genetic information important in genetic testing in hereditary risk. D. RNA to DNA to protein explains the chain of events for protein production.

B. it explains the production of protein for all body functions. "DNA makes RNA and RNA makes protein," explains the production of protein for all body functions". This knowledge aids in the education of patients and families about their inherited genetic testing results. A is incorrect because, while this is an accurate statement, it is not necessary information for oncology nursing practice. D is also false as "DNA makes RNA and RNA makes protein" directs the protein production in the body. C is also false as RNA contains Uracil instead of the Thymine contained in DNA.

An example of how tumor cells can evade the adaptive immune system would include which one of the following? A. becoming resistant to chemotherapy through drug activation B. promotion of T-cell exhaustion by upregulating checkpoint molecules C. the innate immune system keeping the disease in check D. increasing antigens that are recognized by the adaptive immune system

B. promotion of T-cell exhaustion by upregulating checkpoint molecules. Chemotherapy resistance occurs as a result of changes in tumor biology, such as drug inactivation, drug target alteration, drug efflux, DNA damage repair, and cell death inhibition. These are changes that occur with the tumor's interaction with the chemotherapeutic agent and not with the host's immune system. Response B reflects one of many examples of how persistent tumor clones have acquired the ability to "hide" from the immune system, leading to continued reproduction and cancer progression. By promoting T cells to increase PD1/PDL1 on their surface, tumor cells are decreasing T-cell immune surveillance and reproduction, which allows for continued growth of the tumor. Recently developed immunotherapeutic medications, called checkpoint inhibitors, use this known concept of tumor biology to block tumor cells from promoting hosts' T-cell exhaustion, which stimulates the adaptive immune system. The innate immune system is the body's first line of defense against host invaders and works with the adaptive immune system in tumor surveillance. When the innate immune system can no longer keep the tumor in check (equilibrium phase), the adaptive immune system can function independently to perform ongoing immune surveillance. Another method of immune system evasion is decreasing or losing antigens, not increasing, on the tumor's surface. When this occurs, the adaptive immune system can no longer recognize the tumor cell. This leads to ongoing progression/reproduction of cancer.

Molecule found in blood, tissues, or other body fluids that signals the presence of a condition or disease; can be used to evaluate response to treatment; also referred to as molecular marker or signature molecule?

Biomarker

Functions as both primary and secondary lymphoid organ?

Bone Marrow

Location of B-cell differentiation and maturation?

Bone Marrow (BM)

Organ and tissue components of the immune system include primary lymphoid organs and secondary lymphoid organs and tissues. Secondary lymphoid tissues are sites where antigens are captured and processed in the body. Which one of the following is an example of both primary and secondary lymphoid tissue? A. Spleen B. Lymph nodes C. Bone marrow D. Thymus

C. Bone marrow. The bone marrow functions as both a primary and secondary lymphoid tissue. The thymus is a primary lymphoid organ and allows for the maturation of the lymphocytes. The spleen, lymph nodes, and tonsils/adenoids are examples of secondary lymphoid tissue. The spleen responds to bloodborne antigens, while the lymph nodes initiate immune responses to antigens circulating in the lymph, skin, or mucosal surfaces.

A new oncology nurse asks how to determine if a cell is somatic or germline. You explain that: A. Germline cells will have single nucleotide polymorphisms (SNPs) associated with a mutation. B. A somatic cell acquires mutations during the monthly reproductive cycle of the body. C. Germline mutations occur in reproductive cells of a person with an inherited cancer predisposition. D. Somatic cells accumulate mutations prior to conception while in utero.

C. Germline mutations occur in reproductive cells of a person with an inherited cancer predisposition.Germline mutations occur in the reproductive cells of a person with an inherited predisposition to cancer. A is incorrect because single nucleotide polymorphism (SNPs) occur in both germline or somatic cells. A SNP is a change in a nucleotide of a gene causing variation in the DNA sequence that affects 1% of population. B is incorrect as both males and females develop mutations in their somatic cells over a lifetime. Only females have a monthly reproductive cycle. D is incorrect as somatic mutations occur in body cells (except the gametes) after conception and are acquired over a lifetime.

Which of the following may be a psychological consequence to the patient that receives genetic testing results that reveal substantially increased risk for developing cancer or another primary lesion? A. Depression yet a sense of relief B. Transmitter guilt C. Heightened anxiety D. Survivor guilt

C. Heightened anxiety. Heightened anxiety may result when patients learn that they are at a substantially increased risk for developing cancer or another primary lesion. A is incorrect as, although there can be depression, a sense of relief has not been reported; in fact they experience more anxiety. B is incorrect as this relates to family members versus the patient as it occurs when family members pass on the genetic mutation to one of their offspring. D is incorrect as you see this type of guilt in persons who have not inherited the genetic mutation present in other close family members.

Which of the following is a definition of gene amplification? A. Loss of all or part of a gene found in cancer and other genetic diseases. B. Heritable change that does not alter the DNA sequence but changes gene expression. C. Increase in the number of copies of a gene that may cause cancer cell growth or resistance to anticancer drugs. D. Increase in the copies of a protein made from a gene that may play a role in cancer development.

C. Increase in the number of copies of a gene that may cause cancer cell growth or resistance to anticancer drugs. The correct answer is C. Gene amplification is defined as the increase in the number of copies of a gene that may cause cancer cell growth or resistance to anticancer drugs. The loss of all or part of a gene found in cancer and other genetic diseases is defined as gene deletion (answer A). Answer B is epigenetic alteration, which is defined as a heritable change that does not alter the DNA sequence but changes gene expression. And, finally, the increase in the copies of a protein made from a gene that may play a role in cancer development is gene overexpression (answer D).

Lymphoid stem cell lineage has several types of lymphocytes that play a key role in immune responses. Which one of the following cells migrate to the thymus gland for maturation and are integral to immune surveillance and response? A. NK cells B. Cytotoxic T cells C. T cells D. B cells

C. T cells. The T cells migrate to the thymus gland for maturation and are integral to immune surveillance and response. NK cells are large granular cells that release cytokines, migrate rapidly to the site of the inflammation, and directly kill tumor or viral-infected cells without previous antigen exposure. Cytotoxic T cells play a role in autoimmunity and allogenic organ rejection and destroy viral infections and cancers. B cells develop in the bone marrow and include memory B cells and plasma cells.

A patient is being seen in the high-risk cancer clinic because there are features suggestive of a risk for hereditary cancer. These features could include: A. passage of a trait to future generations on only one allele of a chromosome. B. no known germline deleterious mutation in a cancer susceptibility gene. C. autosomal dominant designation of a cancer. D. a unique variety of cancer types in multiple generations.

C. autosomal dominant designation of a cancer. Most hereditary cancer syndromes are inherited in an autosomal dominant fashion. A is incorrect because in most cases the altered gene is passed from one side of the family. B is incorrect as there are deleterious germline mutations in cancer susceptibility genes that are suggestive of risk for hereditary cancer. D is incorrect as cancer types are similar in multiple generations, usually from one side of the family, with a risk for hereditary cancer.

A germline mutation occurs in the gametes and: A. is present only in first generation. B. is present in non-reproductive cells. C. can include a de novo mutation. D. includes every cell in the body.

C. can include a de novo mutation. A de novo mutation is change in a gene and is present for the first time in one family member due to mutation in a germ cell (egg or sperm) of one of the parents or in the fertilized egg. A is incorrect as a germline mutation is passed from generation to generation. B is incorrect as germline mutations are present in the reproductive cells (the eggs and sperm). D is incorrect as it only occurs in the gametes: eggs and sperm.

Recent surveillance imaging has revealed that an oncology patient, who is currently in remission, has developed new lesions. The phase of tumor suppression mechanism most likely to be implicated in this new development can be classified as which one of the following? A. elimination B. equilibrium C. escape D. progression

C. escape. The elimination phase represents ongoing immune surveillance where the host is cancer free. In the situation above, the patient has relapsed. The equilibrium phase is a phase by which a rare tumor clone mutates to avoid elimination by the innate immune system. There is no evidence of clinically measurable disease in this phase. This does not apply to patient situation, as he/she has new lesions. The escape phase correlates with mutated tumor clones which have evaded the innate and adaptive immune systems, resulting in clinically measurable disease, as seen in this example. Progression is not a phase of the proposed tumor suppression mechanism.

A negative genetic testing result with "known family genetic mutation" should include the following caveat: A. the technique used has limited sensitivity. B. the family may be affected by a mutation in another gene. C. family history from the other parent influences the risk of developing cancer. D. the cancer in the family may not be caused by a germline genetic mutation.

C. family history from the other parent influences the risk of developing cancer. When an individual is tested for a known family mutation, they did not inherit the risk associated with the mutation from the side of the family with the known mutation. The history from the other side can also influence risk. A negative test result in a family with a known mutation implies at least population risk for developing malignancy and if there is risk from the other side of the family risk could be increased. A is incorrect as identification of a germline mutation in a cancer susceptibility gene also may not be possible because of the limited sensitivity of the techniques used and this can occur with both a known family mutation and when there is no known family mutation. B and D are incorrect as this can be found with a "no known family genetic mutation," and may offer a reasonable explanation as to why a mutation was not detected in a family with suspected genetic risk.

The use of targeted therapies is common within oncology practice and involves the use of genetic and genomic information of the cancer tissue to guide selection of an appropriate targeted drug therapy. One of the benefits of using targeted therapies is: A. less severe adverse reactions. B. not classified as hazardous drugs. C. spare normal cells. D. only available in oral formulation.

C. spare normal cells. Clinically relevant biomarkers are associated with specific cancers; ideal targets are present in cancer cells but not in normal cells. Targeted therapy has adverse reactions, albeit different from traditional chemotherapy. Targeted therapies can be classified as hazardous drugs, and can be delivered orally (small molecule drugs) or parenterally (Monoclonal Abs).

The use of ER/PR/Her-2/Neu status to guide decisions on the use of adjuvant systemic chemotherapy is an example of: A. use of a cancer-risk assessment tool to confirm suspected familial cancer syndromes. B. dose determination based on genomics. C. use of biomarkers to guide treatment decisions. D. use of testing to determine eligibility for a clinical trial.

C. use of biomarkers to guide treatment decisions. ER/PR/Her-2 Neu are examples of biomarkers used to guide treatment decisions. Germline genetic testing might confirm the presence of a hereditary cancer predisposition syndrome. ER/PR/Her2 Neu status does not alter the dose of the drug but it might alter the choice of agent. Adjuvant chemotherapy decisions might also be based on tools that assess a combination of genes on a tumor to determine the potential efficacy of chemotherapy (e.g. Mammaprint, Endopredict, or Oncotype). Er/PR/Her2Neu status might be one criteria utilized to determine if a patient is eligible for a clinical trial but it will not be the sole criteria.

What is it called when normal cells are transformed into cancer cells through a complex and dynamic process that starts with mutations in regulatory cells and is promoted by genomic instability, inflammation, and interactions within the tumor microenvironment?

Carcinogenesis

Deoxyribonucleic acid (DNA) mutations can be caused by environmental factors known as?

Carcinogens

What are the DNA repair genes that correct mistakes in normal cells that might be caused by carcinogens during replication. In some individuals, these genes may not be functional, which makes it easier for a mutation to result in cancer called?

Caretaker Genes

What results when pieces of one chromosome move to another chromosome as the cell divides. This type of genetic alteration may activate an oncogene.

Chromosome translocation

Which of the following provisions of informed consent for genetic testing is not included when consented for testing? A. Purpose of the test B. Motivation for testing C. Impact of test result on healthcare decision making D. Longevity of life after testing

D. Longevity of life after testing. Genetic testing cannot determine longevity of life so it is not part of the informed consent process. Answers A, B, and C are parts of the informed consent. Elements of informed consent include discussion of the purpose of the test, motivation for testing, risks and benefits of testing, potential limitations of testing, risk of misidentified paternity, inheritance pattern of the gene and likelihood of a mutation being detection, accuracy of the test, potential outcomes of testing, how confidentiality will be maintained, the possibility of discrimination, alternatives to testing, how testing will influence health care decision making, costs of testing and considerations for testing in children.

Which one of the following is a key association and cause of cancer? A. Three servings of fresh fruits and vegetables weekly B. White meat in daily diet C. Too much fiber in daily diet D. Sun exposure and indoor tanning parlors

D. Sun exposure and indoor tanning parlors. A history of sunburns and tanning parlor use is a key association and cause of cancer. Too much fiber in a person's daily diet is not identified as key association and cause of cancer rather lack of fiber is identified as a cause for cancer. Other key association and causes of cancer include lack of exercise and daily intake of processed red meat. Five to nine servings of fruits and vegetables are recommended daily. Lack or limited intake of fruits and vegetables is a key association and cause of cancer.

When administering precision medicine treatments, the healthcare team must be aware of ethical considerations of the patient. Which one of the following is an ethical concern? A. After beginning treatment of a targeted therapy, the patient develops an unexpected adverse event, leading to life-threatening complications that neither the patient nor the patient's family had expected. B. Though the patient has been educated on the topic, the side effects of a particular treatment have severely comprised the patient's quality of life. C. The cost of the treatments is such that the patient experiences financial toxicity and must stop treatment while deciding how to pay the price of drugs. D. The patient is concerned about data security and fear government intrusion that may affect health care coverage.

D. The patient is concerned about data security and fear government intrusion that may affect health care coverage. The correct answer is D. Patients provide biospecimens for biorepository and research purposes in addition to use in clinical decision making, and an ethical consideration for a patient undergoing treatment with precision medicine is whether or not their personal information is safe and secure from cyber crime as well as intrusions from the government. Adverse events and side effects are clinical concerns and not necessarily ethical considerations, and, while financial toxicity is a major concern for patients and families undergoing expensive treatments, the cost of treatment may not be an ethical concern, especially if costs were explained before treatment begins.

R.J. is a 70-year old male whose lung metastases have spread to other regions of his body. He asks the nurse attending to him how the spread of his lung metastases has occurred. The nurse explains to R.J. that the process involves feeding the tumor with oxygen and nutrients that, therefore, allows the tumor to enlarge. The process the nurse has described to her patient is called which one of the following? A. carcinogenesis B. glycolysis C. pathogenesis D. angiogenesis

D. angiogenesis. Angiogenesis is the creation of new blood vessels from existing ones to provide nutrients and remove waste products. Carcinogenesis is the formation of a cancer where normal cells are transformed into cancer cells. Glycolysis is the breakdown of glucose by enzymes, releasing energy and pyruvic acid and is not involved in metastases. Pathogenesis of a disease is the biological mechanism that leads to the disease state, and can describe the origin and development of the disease.

A patient with history of exposures to an environmental carcinogen has come to a class on environmental risk and epigenetics. The audience asks "why/how" that matters. The nurse responds with the understanding that: A. epigenetics causes pieces of the DNA sequence to be "chopped out". B. only epigenetic changes associated with aging, drug use, and addiction cause cancer. C. most cancers are the result of inherited DNA and RNA mutations in single cells over the lifetime of an individual. D. changing the histone structure of DNA by allowing transcription and preventing transcription can alter the protein product outcome.

D. changing the histone structure of DNA by allowing transcription and preventing transcription can alter the protein product outcome. Epigenetics is defined as switching genes on and off with a variety of "chemical tails" attached to the DNA structure, without changing the DNA sequence. DNA transcription is controlled by opening or closing the tightly wound histone structure. This occurs by opening (allowing transcription) and closing the histone structure to alter the "DNA to RNA to protein" outcome. A is incorrect as this statement is incomplete and therefore incorrect. B is also a FALSE statement because any of the environmental effects noted can be positively (prevention) or negatively (causative) associated with development of cancer. C is a FALSE statement. Most cancers a

Pedigree construction that identifies a family at high risk for inherited cancer should include: A. at least four generations of cancer information for both lineages. B. the use of squares to designate females and circles to designate males. C. race, ethnicity, and age of individuals, but only if there is cancer in the generation. D. history of treatments that may have reduced risk of cancer.

D. history of treatments that may have reduced risk of cancer. Any individual with a diagnosis of cancer should provide information about treatments, age at onset, and other pertinent medical history that might explain the diagnosis of cancer such as risk factor exposures. A is an incorrect answer as only three generations of cancer information for both lineages is required. B is incorrect as females are designated as circles while males are designated as squares. C is incorrect as race, ethnicity, and age of individuals should be included for all of the individuals in a 3. Identification of a germline mutation in a cancer susceptibility gene may not be possible because of the limited sensitivity of the techniques used generation pedigree.

The field of pharmacogenomics has revolutionized cancer care because it: A. increased rates of non-adherence to therapy. B. lowered the cost of cancer treatment. C. increased enrollment in clinical trials. D. increased the safety and efficacy of cancer drugs.

D. increased the safety and efficacy of cancer drugs. Pharmacogenomics is the integration of pharmacology and genomics in developing safe and effective medications. Pharmacogenomics can determine safe doses based on genomic data. It can help reduce the use of drugs with serious or toxic side effects thereby increasing rates of adherence to therapy. It does not necessarily decrease the cost of medications or increase enrollment in clinical trials.

An example of humoral immunity is: A. during antigen presentation to naïve T cells by antigen-producing cells (APCs) B. T lymphocyte activation C. the destruction of foreign particles by neutrophils D. the production of plasma cells

D. the production of plasma cells. Antigen presentation refers to the process of activating T cells, which is called cell-mediated immunity. T lymphocytes activation by APCs leads to T-cell multiplication immune surveillance/ response. T lymphocytes are involved in cell-mediated immunity as part of the adaptive immune system. Neutrophils are a part of the innate immune system, not adaptive immune system. Humoral immunity involves antibodies produced by B lymphocytes. Each B cell reproduces and differentiates to become either a memory B cell or plasma cell. Plasma cells then circulate and bind to specific antigens, which starts a cascade of cytokine reactions to attract macrophages and NK cells.

Cancer prediction tools include the following criteria, EXCEPT: A. demographics B. medical history C. family history D. tumor stage

D. tumor stage. Risk assessment tools estimate a person's risk of developing cancer over set period of years (next five years) or over a lifetime. There are tools readily available to provide estimates of the likelihood of developing cancers of the breast, colon, malignant melanoma, prostate, and lung cancers. These risk assessment models combine demographic variables such as age, gender, and ethnicity, medical history such as prior surgeries, presence of colon polyps and reproductive history, family history of malignancy, and lifestyle factors such as tobacco use, diet, and sun exposure.

One example of a medication that hypomethylates is?

Decitabine, which is used in the treatment of myelodysplastic syndrome (MDS). If DNA is not transcribed, it is essentially silenced.

Stage in drug development where a molecular target is evaluated for therapeutic potential; new therapies can be designed to disrupt activity of validated targets?

Drug target validation

Heritable change that does not alter the DNA sequence but changes gene expression?

Epigenetic alteration

What describes a mechanism that may change the activity of a gene without changing the sequence of DNA called?

Epigenetics, an example of an epigenetic mechanism is DNA methylation. Adding or removing methyl groups from DNA affects the transcription of genes. The addition or subtraction of methyl groups may be affected by diet, the environment, and certain medications.

Increase in the number of copies of a gene that may cause cancer cell growth or resistance to anticancer drugs is defined as what?

Gene amplification

Loss of all or part of a gene found in cancer and other genetic diseases is defined as what?

Gene deletion

Increase in the copies of a protein made from a gene that may play a role in cancer development?

Gene overexpression

The study of heredity and the variation of inherited characteristics?

Genetics

Also known as "omics"; the study of the sequence of adenine (A), cytosine (C), guanine (G), thymine (T), or leers, within DNA?

Genomics

Mutations can also be inherited or ______ mutations.

Germline

BRCA1 and BRCA2 are examples of what type of variant?

Germline, when someone inherits one of these germline variants, they become more susceptible to certain types of cancer.

What is the regulation, production, and development of blood cells?

Hematopoiesis

Study of health maintenance and homeostasis based on identification and eradication of foreign microbes, repair of tissue damage, and defense of the host through the capacity to distinguish self from nonself, while preventing malignant proliferation is the definition of what term?

Immunology

1st Step in the development of a cancer is known as?

Initiation; 1st step in the two step theory of causation. Changes to the cell in response to exposure of a carcinogen.

Initiate immune responses to antigens circulating in the lymph, skin, or mucosal surfaces?

Lymph nodes

Gastrointestinal (GI) mucosa (Peyer patches), tonsils/adenoids (Waldeyer ring), and other body systems' mucosa-associated lymphoid tissue (MALT)?

Lymphoid Tissue

What is the spread of cancer cells from the site of the original tumor or organ to distant tissues and organs in the body?

Metastasis,

Therapeutic agents that may target more than a single biomarker?

Multitarget inhibition

Also known as high-throughput sequencing; a term used to describe modern DNA and RNA sequencing technologies?

Next-generation sequencing

Most DNA mutations are ____, also known as ____.

Noninherited, somatic mutations

Analysis of a tumor or other tissue to identify genomic, proteomic, or metabolomic alterations of clinical importance?

Omic profiling

Exploration of the roles, relationships, and actions of the molecules that comprise the organism; includes genomics, proteomics, and metabolomics?

Omics

Receiving personalized care or developing patient-specific drugs, Ability to provide therapies specifically designed for an individual, Tailoring of medical treatment to the individual characteristics of each patient is described as what?

Personalized Medicine

Looks at how the drug affects the individual?

Pharmacodynamics

Study of how patient's genomes affect responses to medications?

Pharmacogenomics

Looks at how the individual affects the drug is defined as what?

Pharmacokinetics

What is described as the use of specific information about a person's genes, proteins, and environment to prevent, diagnose, and treat disease. It Classifies individuals into subpopulations based on susceptibility to a particular condition, cancer biology, disease prognosis, and response to specific treatment. It allows for preventive or therapeutic interventions to be delivered to individuals more likely to benefit, sparing those who will not benefit from the cost and toxicities of ineffective treatment. Not to be confused with personalized medicine.

Precision Medicine

A biomarker that provides information on the effect of a therapeutic intervention is what?

Predictive biomarker

Allow for the maturation of lymphocytes and includes the following organs: a. Bone marrow (BM)—location of B-cell differentiation and maturation b. Thymus—location of T-cell differentiation and maturation

Primary Lymphoid Organs

A biomarker that provides information about the patient's overall cancer outcome, regardless of therapy is defined as what?

Prognostic biomarker

3rd step in the development of a cancer is known as?

Progression; The transformation from a benign to malignant neoplasm. The changes that have occurred in the cells are characterized by increased growth rate, invasion, and ability to metastasize.

2nd step in the development of a cancer is known as?

Promotion; Occurs when the cell that has been initiated develops specific changes and can become malignant. Often requires long exposure to carcinogen and may be reversed.

What are the genes that code for proteins involved in normal cell growth, and when mutated, they may enable a cancer cell to be self-sufficient in growth called?

Proto-oncogenes

L.S. is a 49-year-old female with pancreatic cancer. During the course of her diagnosis and treatment, L.S. asks the oncology nurses in her care about the cause of her cancer. The oncology nurse responds to the question from L.S. with the understanding that the gene mutation frequently affecting this condition is caused by which one of the following: A. a missense gene mutation B. a chromosome translocation C. a proto-oncogene mutation D. an insertion mutation

Proto-oncogenes that are mutated, such as Ras, can enable a cancer cell to be self-sufficient in growth, and are common in pancreatic and colorectal cancers. In chromosome translocations, one chromosome moves to another as the cell divides, thereby activating an oncogene, such as occurs in CML where the BCR gene on chromosome 9 is fused to the Abl gene on chromosome 22, making a protein called tyrosine kinase, which proliferates myeloid cells. Missense mutation changes a DNA base pair that results in the substitution of one amino acid for another in the protein made by a gene and is typically the cause of sickle cell disease. An insertion mutation is the addition of one or more nucleotide base pairs into a DNA sequence, such as occurs in Huntington's disease or fragile X syndrome.

What proto-oncogene is commonly mutated in pancreatic and colorectal cancers?

RAS, a point-mutation that happens at a single location of DNA or single base pair, that can change proto-oncogene to an oncogene.

Sites where antigens are captured and processed a. Lymph nodes—initiate immune responses to antigens circulating in the lymph, skin, or mucosal surfaces b. Spleen—responds to bloodborne antigens c. Bone marrow—functions as both primary and secondary lymphoid organ d. Lymphoid tissue—gastrointestinal (GI) mucosa (Peyer patches), tonsils/adenoids (Waldeyer ring), and other body systems' mucosa-associated lymphoid tissue (MALT)

Secondary lymphoid organs and tissue

Transforming growth factor alpha (TGF-α) or the epidermal growth factor receptor (EGFR) signaling pathway plays a role in...?

Some colon cancer metastases. Some metastatic colon cancer cells produce five times more EGFR compared with nonmetastatic cells.

Responds to bloodborne antigens?

Spleen

Which proto-oncogene is normally located on chromosome 8. In the Burkitt lymphoma cells, this portion of DNA is relocated to chromosome 14?

The MYC proto-oncogene

In chronic myeloid leukemia (CML), the BCR gene on chromosome 9 is fused to the Abl gene on chromosome 22. This translocation is also known as....

The Philadelphia chromosome. This fusion makes a protein called a tyrosine kinase, which promotes proliferation of myeloid cells. Imatinib, the original targeted tyrosine kinase inhibitor, was designed to specifically inhibit this pathway.

What happens when a mutation occurs and tumor suppressor genes are inactivated?

They become inactive and are unable to stop cell division.

Location of T-cell differentiation and maturation?

Thymus

Which type of gene promotes cell apoptosis?

Tumor Suppressor Gene; genes that control proliferation by preventing uncontrolled growth. When mutated, these genes no longer suppress proliferation.

Genes that control proliferation by preventing uncontrolled growth. When mutated, these genes no longer suppress proliferation.

Tumor Suppressor Genes

What protein promotes proliferation of myeloid cells?

Tyrosine Kinase


Ensembles d'études connexes

practice quiz - great depression 1920 - 1940

View Set

FIN 3403 - CH 8 - Stock Valuation

View Set